Você está na página 1de 51

Acute Surgical Abdomen

BACKGROUND
This 42-year-old man presents to the emergency department after approximately 30 hours of abdominal pain. The patient's pain was initially mild,
constant, and periumbilical, lasting nearly 10 hours before resolving. After he ate breakfast, the pain returned to the right lower quadrant and gradually
progressed over the next 20 hours.
The patient denies any fever, nausea, vomiting, diarrhea, constipation, bloody stools, weight loss, testicular pain, or anorexia. Physical examination
reveals an afebrile, well-appearing man with localized voluntary guarding and rebound tenderness in the right lower quadrant. In addition, he has
positive heel tap, Rovsing, and psoas signs. Contrast-enhanced CT of the abdomen and pelvis is performed (see Image A).
Hint
The patient denies ingesting a foreign body, but he cannot tolerate having food stuck between his teeth.
Answer
Unknown toothpick ingestion with perforation of the cecum: Although the patient^s presentation is classic for appendicitis, he lacks some findings on
review of systems. He was in no distress and afebrile, without any anorexia or vomiting. Initial CT of the abdomen showed a thickened terminal ileum; a
normal-appearing appendix; and perimesenteric fat stranding, which was read as nonspecific inflammation of the bowel most consistent with regional
enteritis. However, because his pain worsened over the next several hours, he was taken to the operating room for diagnostic laparoscopy. As shown
on the intraoperative image (see Image B), a toothpick was found perforating the cecum. The toothpick was removed, and the patient had an
uncomplicated recovery.
Toothpick ingestions are rare, but the literature includes several case reports. Approximately 70% of patients with reported toothpick ingestions present
with abdominal pain. However, only about 12% remember ingesting the toothpick. The onset of symptoms varies, with a reported range of less than 1
day to 15 years after ingestion.
Perforation most frequently occurs at the duodenum and sigmoid, but this case shows that perforation may occur anywhere. Imaging studies are useful
in only 14% of cases; laparotomy is the most common method for definitive diagnosis. The overall reported mortality rate is as high as 18% (Li, 2002).
As a general rule, patients ingesting sharp objects and objects larger than 2 X 5 cm should be watched closely and treated aggressively.
This case reminds physicians that a CT may be bypassed in a patient with a surgical abdomen. Although the patient lacked some classic symptoms of
appendicitis, the history and examination findings were consistent with a surgical abdomen. CT may have delayed the appropriate treatment, which was
diagnostic laparoscopy.
For more information on foreign body ingestions, see the eMedicine articles Foreign Bodies, Gastrointestinal, Gastrointestinal Foreign Bodies, and
Pediatrics, Foreign Body Ingestion.

Acute Tongue Swelling

BACKGROUND
A 65-year-old Hispanic woman presents to the emergency department because of tongue swelling, difficulty swallowing, and vocal changes that have
lasted 3 hours. She reports having several mild episodes in the past that resolved spontaneously at home. She has a history of hypertension, asthma,
and diabetes mellitus. She denies any known drug allergies or recent changes in her medications. She denies having any exposure to new lotions,
soaps, detergents, or perfumes. She denies having any fever, chills, recent dental work, or trauma to the tongue.
Physical examination reveals morbid obesity and normal vital signs. The patient is sitting upright on a stretcher and leaning forward, drooling, and easily
ventilating. Her voice is muffled. She has marked, nontender edema of the tongue and the floor of the mouth but not on her lips or face. No erythema,
increased warmth, or induration is observed.
An emergency airway tray is placed at her bedside as an otolaryngologist performs nasopharyngolaryngoscopy; this does not reveal any edema of the
uvula, hypopharynx, or vocal chords. No stridor is noted, and her lungs are clear to auscultation. The patient is given intravenous Solu-Medrol,
famotidine, and diphenhydramine and admitted to the ICU. The swelling gradually improves, and no airway interventional methods are needed.
Hint
The patient is taking several medications, including one for hypertension.
Answer
Angiotensin-converting enzyme inhibitor (ACEI)?induced angioedema: Angioedema is a potentially life-threatening adverse effect of ACEI therapy, one
characterized by well-demarcated swelling of the lips, face, and oropharynx (Gainer, 1996). The angioedema most commonly affects the head and
neck, but the viscera can be involved as well. In some cases, ACEI therapy causes localized angioedema affecting the tongue alone (Kyrmizakis, 1998).
The estimated incidence of angioedema during ACEI treatment is 0.1-0.2%. Approximately 60% of cases occur during the first week of treatment,
though several cases have been reported after several years. The incidence is up to 3 times greater in certain groups such as African Americans, in
whom low levels of endogenous bradykinin heighten their sensitivity to bradykinin-related changes during ACEI therapy (Gainer, 1996).
Kininase II is the main inactivator of bradykinin and identical to angiotensin-converting enzyme (ACE). By blocking both ACE and kininase II, ACEIs
inhibit the metabolism of bradykinin, a potent vasodilator and mediator of capillary leakage. Increased tissue levels of bradykinin are thought to induce
angioedema. Other immunologic processes and mediators, such as histamine, substance P, and prostaglandins, may also be involved. Evidence to
support this mechanism includes the increased incidence of ACEI-induced angioedema in patients with ACE-gene polymorphism and enzyme
deficiencies (Vleeming, 1998).
Standard management includes airway protection and supportive treatment with antihistamines and/or corticosteroids, although no controlled studies
have confirmed their efficacy (Vleeming, 1998). The clinician must realize that the use of standard allergic therapy (eg, antihistamines, corticosteroids)
may not rapidly produce a significant positive effect. This blunted response to standard allergic therapy results from the pathophysiologic basis of
bradykinin as the primary mediator rather than histamine. In life-threatening angioedema, epinephrine and early intubation or cricothyroidotomy may be
necessary. Prophylactic endotracheal intubation may be required for airway protection. Fresh frozen plasma (FFP) may be beneficial in unresponsive
cases of severe ACE-I angioedema. However, FFP treatment is associated with a risk of allergic reaction, volume overload, and the transmission of viral
disease. In addition, FFP is beneficial in hereditary angioedema, in which it replaces the deficient C1 esterase inhibitor. FFP also provides additional
kininase II, which breaks down accumulated bradykinin, and it provides functional enzyme to patients with enzyme defects in bradykinin metabolism
(Karim, 2002).
For more information on angioedema syndrome, see the eMedicine articles (Medicine) Angioedema and (Emergency Medicine) Angioedema.

An Allergen and Two Innocent Bystanders

BACKGROUND
A woman presents to your office concerned about the plants in her wildflower patch. She believes that one of the plants (see Image) is causing
"allergies." She complains, "Each September when I tend to these flowers, my eyes water, my nose runs, and my asthma worsens." A bit uncomfortable
with your expertise in botany, you ponder the potential diagnoses, recall the most common causes of seasonal allergies, and prepare to offer advice.
What is the diagnosis?
Hint
One of the plants is a common cause of seasonal allergies.
Answer
Seasonal allergies due to ragweed: After examining the flowers, you recognize the bloom with the small flowers (middle of the image) as a chive, Allium
schoenoprasum. The plant with the bright yellow blossoms (right) is goldenrod, Solidago species, and the remaining plant with greenish yellow, bellshaped flowers (left, 10-o'clock position) is ragweed, Ambrosia artemisiifolia.
Chives grow in clumps to 6-12 inches high. They form a semisphere of purple, pink, or white flowers. Chives are used for cooking but are not allergenic.
Goldenrod blooms from late summer through mid fall. Its bright yellow flowers attract insects and birds. Because the pollen grains are heavy, bees,
butterflies, birds, and ants are needed to pollinate this plant. Goldenrod and ragweed bloom at the same time of year and commonly grow side by side.
Despite its vibrant color and attraction by insects and birds, goldenrod is not responsible for seasonal allergies. Named for the shape of its ragged
leaves, ragweed is a common source of seasonal allergies. Unlike goldenrod, ragweed requires the wind to release pollen and initiate fertilization;
therefore, its flowers are not as brilliant as those of the goldenrod plant. Ragweed is a common cause of allergy symptoms from late summer to mid fall
(August through November). Avoiding ragweed pollen is the most effective method for decreasing ragweed allergy. Closing windows in the car and
home minimizes exposure to the pollen. Decreasing time spent outdoors, especially on dry and windy days, can be helpful as well.
Allergic rhinitis is often caused by exposure to environmental or occupational factors, including perennial allergens (eg, dust mites, mold, grains, wood
dust, latex) and seasonal allergens, mainly tree, grass, and weed pollen such as ragweed, as in this case. Typical physical findings include allergic
shiners, or dark circles around the eyes; swollen and boggy nasal turbinates; and cobblestoning of the posterior oropharynx due to exuberant lymphoid
tissue.
Management of allergic rhinitis consists of both allergen avoidance and pharmacologic management. Environmental control includes avoiding the
specific allergen to which the patient has immunoglobulin E (IgE)mediated hypersensitivity, reducing outdoor exposure during specific seasons,
cleaning indoor environs, and avoiding nonspecific triggers such as smoke, strong perfumes, and fumes.
Treatment with oral antihistamines and decongestants is the cornerstone of the pharmacologic management of allergic rhinitis. Because of their
favorable adverse-effect profile, newer second-generation, nonsedating antihistamines, such as cetirizine (Zyrtec), fexofenadine (Allegra), and
loratadine (Claritin), are preferred over older first-generation antihistamines, such as diphenhydramine (Benadryl) and chlorpheniramine (ChlorTrimeton). In addition, many commercial formulations of the second-generation antihistamines include a decongestant for complete symptom relief.
Nasal steroids can be used to control chronic symptoms; initiating these steroids 1-2 weeks before the beginning of ragweed season offers the best
results. Other symptomatic agents, such as vasoconstrictors (eg, oxymetazoline hydrochloride), provide fast-acting relief of nasal drainage and
congestion. However, these products cannot be used for more than 3 days without a risk of rebound symptoms. Immunotherapy and systemic steroid
treatment may be required in extreme cases of allergic rhinitis, including ragweed allergy.
For more information on seasonal allergies, see the eMedicine articles Allergic Rhinitis (within the Internal Medicine Specialty) and Allergic Rhinitis
(within the Ear, Nose, and Throat specialty).

Anaphylaxis in the Hairdresser's Chair

BACKGROUND
A 37-year-old woman who works as a secretary in a medical and dental clinic is referred for an allergy evaluation. She reports an episode that occurred
3 months ago while she was having her hair styled by a hairdresser. The relaxer, shampoo, and conditioner treatments had been applied without
adverse effect. The hairdresser was placing hair extensions and was beginning to apply weaving bond and lotion to remove the hair bond. Within 5
minutes, the patient began feeling flushed, lightheaded, and short of breath. She developed a cough, neck urticaria, and a sensation of swelling in her
throat.
The event had occurred about 3 hours after the patient ate lunch. Her lunch contained shrimp, which she has since ingested without adverse effect. She
was in her usual state of health on that day and denies having taken medications besides Synthroid (levothyroxine sodium) for hypothyroidism. The
hairdresser was not using latex gloves during the hair appointment.
The patient denies having had swelling of her lip or tongue, peripheral angioedema, a change in vocal quality, and GI symptoms. She was taken to the
emergency department, where she became hypotensive, lost consciousness, and was successfully treated with epinephrine and steroids.
Hint
The woman reports a history of immediate localized pruritus but no systemic symptoms when she uses household gloves while performing cleaning
chores at home.
Answer
Latex hypersensitivity: Weaving bond can contain natural rubber latex (NRL), an antioxidant, a preservative, and color. Contents of bond removers
include water, adepsine oil, carbormer, surfactants, vitamin E, aloe vera, panthenol, propyl and methyl paraben, and fragrance.
In this patient, skin-prick tests reveal a 4+ reaction to the weaving bond and histamine control and a negative response to the hair bond remover and
glycerin control. Three volunteer subjects had negative results to skin-prick testing with the weaving bond. On further evaluation, a serum latex
immunoglobulin E (IgE) radioallergosorbent test (RAST) demonstrated a 938% response (class 4, very high level) in this patient.
Latex, or NRL, is secreted from the rubber tree Hevea brasiliensis. An estimated 5-15% of the population have IgE-mediated sensitivity to NRL. In
particular, the prevalence of sensitization is high among healthcare workers; this rate is related to both their frequent use of latex gloves and their high
workplace exposure to ambient latex allergen. Because powder-free latex gloves can significantly decrease ambient levels of this allergen, their
widespread use is now recommended.
Individuals allergic to latex should avoid latex exposure, wear a MedicAlert bracelet, and carry an EpiPen. Patients with latex allergy can find
educational materials at the Web site of the American Latex Allergy Association. This site also features latex-free products used in hospitals, on crash
carts, in schools, in dentists' offices, and in everyday life by consumers.
For more information on latex allergy and anaphylaxis, see the eMedicine articles Latex Allergy and Anaphylaxis.

Arm Injury After a Motor Vehicle Collision

BACKGROUND
This patient presented to the hospital after being involved in a low-speed motor vehicle collision that caused moderate damage to the car. The patient
experienced no loss of consciousness and was not short of breath. Furthermore, the patient claims that he was driving only 30 miles per hour. What was
the mechanism of injury?
Hint
The car was equipped with the latest safety features. The patient complains of painful burning in his arm.
Answer
Left forearm abrasion and chemical burn secondary to airbag deployment: The patient sustained a significant airbag abrasion and chemical burn.
Airbags deploy in 50 milliseconds at a speed of more than 140 miles per hour. The deployment generates a tremendous amount of force and makes
friction burns possible. Chemical burns are attributed to sodium hydroxide in the aerosol that is created during deployment. Airbags are designed to
deploy in collisions at speeds as slow as 15 miles per hour; therefore, damage to the auto may be minor.

Blurry Vision and Lump on the Right Eyelid

BACKGROUND
A 53-year-old man visits his primary care physician because of blurry vision in his right eye that has lasted approximately 3 weeks. Incidentally, he
mentions that a painless lump has been present on his right upper eyelid for approximately 2 months. What is the diagnosis?
Hint
His visual disturbance varies each hour and sometimes interferes with his ability to read.
Answer
Chalazion of right upper eyelid: A chalazion, or conjunctival granuloma, is caused by an obstruction of 1 of the approximately 40 meibomian glands in
the upper or lower eyelids. The walls of the clogged gland expand and then thicken, ultimately forming a granulomatous nontender mass. This mass
may compress the eye, changing the corneal curvature. Visual disturbances result, which tend to vary during the day because of subtle changes in the
size of the chalazion and the amount of edema in the eyelid. Once the lid lesion is stable, changing the prescription should improve the vision. The
application of warm compresses to the eyelid sometimes reduces the size of the chalazion and improves the vision. If no response occurs, surgical
removal of the granuloma through the inside of the eyelid restores normal vision.

Bump on the Head

BACKGROUND
A 14-year-old female adolescent presents with a small red spot on her scalp. Her primary care physician advises her to use an antidandruff shampoo.
Over the next 6 days, the lesion increases in size and becomes tender. The family notes that the lesion is crusted and occasionally oozes thick material.
The patient is systemically ill with a fever and poor appetite. Her primary care physician diagnoses a bacterial abscess of the scalp, institutes therapy
with oral dicloxacillin, and requests a consultation with a surgeon for incision and drainage. What is the diagnosis?
Hint
The patient's parents are dairy farmers. Presently, the cows are experiencing an outbreak of a skin disease.
Answer
Kerion (tinea capitis): A kerion is an inflammatory response, likely an allergic reaction, to tinea capitis. Clinically, kerions are boggy and tender scalp
swellings associated with hair loss and purulent drainage. Trichophyton and Microsporum species are the most common fungi that cause tinea capitis.
In more than 90% of cases, tinea capitis is spread between people who share brushes, combs, barrettes, hats, or pillows; fewer than 10% of cases are
transmitted from infected animals. In this patient, Trichophyton verrucosum from the dairy cows was isolated.
Therapy for a kerion includes the use of oral antifungal agents, systemic steroids, and antidandruff shampoo. The antifungal of choice is griseofulvin
ultramicrosize 7.3 mg/kg/d administered orally for 6-8 weeks. When taken with fatty foods, griseofulvin is absorbed effectively. Systemic steroids
decrease inflammation and hasten recovery. Antidandruff shampoo decreases the transmission of tinea capitis between persons in close contact.
Surgical drainage is contraindicated.

Cat Bite to Forearm

BACKGROUND
This 19-year-old pet groomer presents to the hospital 9 hours after being bitten by a cat. She complains of pain and swelling in her left forearm. The
patient also has scratches on the dorsum of her right hand. What tests should be performed? What infectious agent is the most worrisome, and how is it
treated? Should this patient be admitted to the hospital?
Hint
The rapid progression of pain and swelling within hours is a classic sign of this disease. Hand infections should be treated cautiously. This patient has
significant pain with passive extension of her left wrist.
Answer
Pasteurella multocida infection: Pasteurella multocida infection is a forearm cellulitis due to Pasteurella multocida. This organism is found in 53-80% of
cultured infections related to cat bites. Factors that contribute to a poor prognosis include old age, attempted wound care at home, deep puncture
wounds, and delayed treatment. Other risk factors include hand wounds, prosthetic joints, and immunocompromise.
The treatment of a cat bite is essentially the same as that for dog bites. Wound cleansing and wound closure are indicated for most large wounds. The
tetanus status of the patient should be verified. Puncture wounds and lacerations smaller than 1 cm should not be closed because they cannot be
adequately cleaned; however, in cosmetically important areas, delayed primary closure of such small wounds may be performed.
Because the teeth of cats are long and thin, they can potentially break off and become embedded in the wound. A radiograph of the forearm of this
patient did not show a foreign body. This patient was treated with intravenously (IV) administered Unasyn and admitted to the hospital for observation
and continued IV antibiotics. Patients allergic to penicillin could receive erythromycin instead. Reference: Tintinalli JE, Ruiz E, Krome RL, eds.
Emergency Medicine: A Comprehensive Study Guide. 4th ed. McGraw-Hill; 1995:321.

Discharge, Tearing, and Discomfort in the Left Eye

BACKGROUND
A 45-year-old man visits his primary care physician because of discharge, tearing, and discomfort in the left eye that has lasted 3 days. He has no
history of trauma. Examination of the left eye reveals conjunctival edema and vascular congestion with stringy discharge. When fluorescein dye is
placed in the eye, no areas of staining are found, but a number of white spots appear in the superior aspect of the cornea. What is the diagnosis?
Hint
For 3 days, the patient has woken up in the morning with the eyelids of his left eye stuck together.
Answer
Staphylococcal microallergic keratoconjunctivitis: External ocular infection with Staphylococcus organisms sometimes appears as peripheral infiltrates in
the corneal stroma. These white-gray circular areas represent an antigen-antibody mechanism, which is due to the eye^s immune response to
staphylococcal exotoxin. Because these infiltrates are peripheral, they do not compromise the patient^s visual acuity, and severe complications are
unusual in immunocompetent individuals. The condition may be unilateral or bilateral.
Treatment of this condition consists of the application of a fluoroquinolone ophthalmic ointment 3 times a day. The addition of topical steroid drops 3
times a day, preceding instillation of the ointment, causes the infiltrates and symptoms to resolve more rapidly.

Double Vision in an Adolescent

BACKGROUND
A 14-year-old male adolescent visits his primary care physician because of double vision that has lasted 10 days. He says that he sees 2 images that
are vertically separated and that the distance between the 2 images increases as he looks upward. The double vision disappears when he looks down
or closes either eye. No associated neurologic signs or symptoms are present. Examination reveals that the left eye does not move up when the patient
attempts to look upward. What is the diagnosis?
Hint
About 2 weeks ago, the patient was hit in the left eye with a softball, and he had some bruising of the eyelids.
Answer
Blowout fracture of the left orbital floor with entrapment of the left inferior rectus muscle: When blunt force is applied to the eye, the orbital contents are
pushed posteriorly, and the intraorbital pressure suddenly increases. The bony orbital floor, which is also the roof of the maxillary sinus, is thin and may
fracture as a result of this increased pressure. A few milliseconds after impact, as the orbital floor fractures, the edges of the fracture briefly separate
and then come together. At the moment of separation, the contents within the lower part of the orbit may prolapse into the separated fracture space, and
the herniated structures may then become trapped as the pieces come together.
If the inferior rectus muscle is trapped between the edges of the fracture, the patient cannot look upward in a normal fashion. The eyes may be aligned
on downward gaze, but attempted elevation of the eyes causes double vision with the vertical separation of images. In addition, if a significant amount of
orbital fat is caught within the fracture, the eye may appear sunken.
Clinically, the extraocular movements resemble those seen in superior rectus paresis. However, isolated superior rectus palsies are extremely rare.
Patients with significant orbital floor fractures always have hypesthesia in the distribution of the infraorbital nerve because the location of this nerve
within the orbital floor causes stretching or tearing of the nerve fibers. Asking the patient if his or her upper lip is numb on the side of the injury elicits a
positive response, as well as a measure of respect for your diagnostic acumen.

Ear Swelling

BACKGROUND
A 40-year-old man presents with swelling of his right pinna that he noticed upon waking. The patient does not have pain, discharge from the ear, or a
history of trauma or an insect bite to the pinna. He had similar symptoms in his left ear 7 years ago but did not seek medical attention. Examination
findings include cystic, fluctuant, nontender swelling on the lateral part of the right pinna. No insect bite mark or injury to the overlying skin is apparent.
The patient's external auditory canal is normal. What is the diagnosis?
Hint
The man has a noninfective and nontraumatic condition.
Answer
Seroma of the right pinna: Seroma of the pinna is a collection of serous fluid between the perichondrial layer of the pinna and the underlying cartilage.
The etiology is usually unclear, and the condition tends to recur. Treatment consists of needle aspiration or drainage of the fluid with application of a
compression bandage to the pinna. Drainage is necessary to prevent necrosis of the cartilage. Some physicians advocate therapy with a short course of
oral steroids.

Elbow Pain

BACKGROUND
This 19-year-old man was playing basketball. He jumped up to dunk the ball, but he missed and fell on his outstretched hand. He complains of pain in
his right elbow and is unable to extend it fully. Soft tissue swelling is noted in the elbow. What is the diagnosis? How is it managed?
Hint
It definitely is out.
Answer
Posterior elbow dislocation: At least 80% of elbow dislocations are posterior, caused by hyperextension of the elbow and axial compression. Fracture of
the coronoid process occurs in as many as 15% of elbow dislocations. Open reduction and internal fixation (ORIF) of the coronoid process is rarely
necessary, but longer immobilization may be recommended. The neurovascular status should be assessed before the injury is reduced. Conscious
sedation is recommended in patients who present for care more than 1 hour after the injury occurs because muscle spasm develops rapidly. Morphine
and midazolam were administered to this patient. Regardless of whether 1 or 2 persons are involved in the reduction, gentle traction should be applied
to the forearm with countertraction on the brachium. Afterward, the elbow should be moved through its full range of motion to ensure that no block to
motion exists and to document any instability. Instability requires the application of a posterior mold, whereas a stable joint can be treated with only a
sling. Follow-up with an orthopedic surgeon should be arranged. Reference: Browner BD, Jupiter JB, Levine AM, et al. Skeletal Trauma. 2nd ed.
1998:1474-1482.

Fall From 25 Feet

BACKGROUND
This young man fell from a height of approximately 25 feet. He sustained open tibial and fibular fractures in his left lower extremity. He has no neck pain
or other obvious fractures. What is the diagnosis? What other fractures should be considered? What radiographs should be obtained?
Hint
Think about the mechanism of injury.
Answer
Open tibia-fibula fracture: Calcaneal fractures should be suspected, and specific calcaneal radiographs should be ordered. As many as 75% of
calcaneal fractures include articular disruptions and result in significant complications, such as posttraumatic arthritis and chronic pain with walking. In
addition to lower extremity fractures, pelvic and spinal fractures should be suspected. Falls should be treated according to advanced trauma life support
(ATLS) protocols. The median lethal dose (LD50) for falls is 4 stories, or 48 ft, and the lethal does for 90% of test subjects (LD90) is 7 stories, or 84 ft.
Reference: Rosen P, ed. Emergency Medicine: Concepts and Clinical Practice. 4th ed. Mosby-Year Book, Inc; 1998:352.

Fall on Outstretched Hand

BACKGROUND
A woman presents to the hospital with pain in her right wrist caused by a fall on her outstretched hand. On examination, swelling and tenderness are
noted in the distal forearm. The classic dinner-fork deformity is recognized.
Hint
Fracture types are difficult to memorize. However, this fracture has several types; the most important of these to recognize is IIA because anything more
serious requires immediate referral to an orthopedic surgeon. Visualizing the anatomy is helpful in identifying possible associated injuries.
Answer
Colles fracture: A Colles fracture is an extension injury of the distal forearm. The basic types are classified according to their involvement, as follows: I,
extra-articular; II, radiocarpal joint; III, radioulnar joint; and IV, radiocarpal and radioulnar joints. Each type is further classified as subtype A if it involves
a radial fracture and as subtype B is it involves an ulnar fracture. For example, a type IIA Colles fracture is a distal radial fracture with radiocarpal joint
involvement. The normal radiocarpal joint is angled 23? in the ventral direction. Dorsal angulation of the radiocarpal joint results in poor functional
recovery with inadequate reduction. Approximately 60% of all distal radial extension fractures are accompanied by ulnar styloid fractures, and 60% of
ulnar styloid fractures are associated with fractures of the ulnar neck. Carpal fractures, distal radioulnar subluxation, flexor tendon injuries, and median
or ulnar nerve injury may be other associated injuries. Abnormal median nerve function requires an assessment of the pressure in the carpal canal
pressures to differentiate acute carpal tunnel syndrome from median nerve contusion. Type IIA or less serious Colles fractures can be managed with
dorsal and palmar long arm splints with the wrist in 15? of flexion and with 15? of ulnar deviation. Prompt orthopedic follow-up care is necessary. All
other fractures require consultation with an orthopedic surgeon on an emergency basis. Reference: Simon R, Koenigsknecht S. Emergency
Orthopedics: The Extremities. 3rd ed. Norwalk, Conn: Appleton & Lange; 1996:136-42.

Foreign Body Sensation in the Eye

BACKGROUND
A 35-year-old woman visits her primary care physician because she awoke with a severe foreign body sensation in her right eye. The foreign body
sensation has been present almost every morning for a week, but, until today, it usually disappeared in about an hour. She denies any history of recent
trauma. Her right eye is sensitive to light and tearing excessively, and she cannot keep it open for examination. However, after a drop of topical
anesthetic and some fluorescein dye is administered in her eye, she is able to open it. Examination with a penlight and blue filter reveal a staining area
in the cornea just superior to the pupil. What is the diagnosis?
Hint
The woman states that she had a similar pain almost 3 years ago when her infant son scratched her cornea.
Answer
Recurrent erosion of the cornea: This common condition results from an injury to the basement membrane beneath the corneal epithelium. Recurrent
corneal erosions typically occur after a corneal abrasion caused by, eg, a fingernail or coniferous branch. A few weeks to a few years later, a mild
foreign body sensation in the previously injured eye occurs when patients first open their eyes in the morning. After a few days with these symptoms,
patients wake up with a severe foreign body sensation and other symptoms similar to those of a new corneal abrasion. The initial injury has often been
forgotten.
The pathophysiology of this condition most likely involves the deposition of molecular material from the initial injury at the level of the basement
membrane (Bowman membrane) of the corneal epithelium. As a result, the epithelial cells fail to adhere normally to the basement membrane. Usually,
tears are slightly hypotonic when the eyes are closed in sleep, and the corneal epithelial cells swell during the night. In cases of recurrent corneal
erosion, these swollen cells slip off the damaged basement membrane when the eyelid is first opened in the morning.
Treatment involves the standard care for a corneal abrasion, which consists of the application of a topical nonsteroidal anti-inflammatory drug (NSAID)
and an antibiotic steroid ointment in the office and no patching. The patient should use combination antibiotic-steroid eye drops 4 times a day for 7 days
and a hypertonic saline (5% NaCl) ointment before sleep for at least a month. This ointment increases the tonicity of tears and compacts the surface
cells, allowing them to maintain better adhesion to the underlying basement membrane. If medicinal therapy is not effective, 3 surgical options exist. The
first is to surgically debride the epithelium and to place a therapeutic contact lens in the eye to allow proper healing of the epithelium. The second is to
use a 30-gauge needle to perform micropuncture of the cornea in the area of the recurrent corneal erosions. The third is to use an excimer laser in the
phototherapeutic keratectomy (PTK) mode in the area involved.

Gynecology in the Upper Abdomen

BACKGROUND
A 33-year-old gravida 5 para 3 woman presents to the hospital because of abdominal pain in the left lower quadrant that has lasted 6 hours. The
woman's last menstrual period was 6 weeks ago. Her past surgical history includes bilateral tubal ligation 4 years ago. The results of a urine pregnancy
test are positive. The b?human chorionic gonadotropin level is 9132 mIU/mL. Transvaginal ultrasonography reveals an empty uterus. Ectopic pregnancy
is diagnosed, and laparoscopy is performed, revealing an unruptured ectopic pregnancy (see Image 1). Laparoscopic left salpingectomy is performed.
On exploration of the upper abdomen, Image 2 is obtained. What is the diagnosis?
Hint
This condition has the same etiology as the tubal pregnancy.
Answer
Fitz-Hugh and Curtis syndrome: In 1920, Stajano first described perihepatitis followed by adhesions between the liver and parietal peritoneum. FitzHugh and Curtis further described this association of perihepatitis with acute salpingitis in 1930. A past episode of acute salpingitis and perihepatitis are
responsible for the classic violin-string adhesions depicted in Image 2. In the acute phase, the patient may have right upper quadrant pain. The
appropriate antibiotics for managing acute salpingitis are aimed at treating the perihepatitis. After the acute phase, most patients have no right upper
quadrant symptoms.
This case also demonstrates the fact that one third of pregnancies occurring after a tubal ligation are ectopic.
For more information about Fitz-Hugh and Curtis syndrome, see the eMedicine article Fitz-Hugh-Curtis Syndrome.

Head Rash

BACKGROUND
A mother brought her young son to the hospital because the rash that she noticed on the day before had significantly worsened overnight. The rash
started on the boy's hands, including his palms, and quickly spread to his arms and head. The boy had upper respiratory symptoms, which lasted 3
days. Now, blisters are present on his oral mucosa. Although the lesions are not pruritic, the boy complains of a burning sensation in affected areas.
What is the diagnosis? What is the prognosis for this patient?
Hint
This condition is on one end of the spectrum of diseases that include Stevens-Johnson syndrome.
Answer
Erythema multiforme (EM) minor: This is the typical iris or target lesion of EM minor that is characterized by bright red borders and central petechiae,
vesicles, or purpura. Most cases are self-limited; lesions evolve over 1-2 weeks and subsequently resolve within 2-3 weeks. Postinflammatory
hyperpigmentation or hypopigmentation may occur. Recurrence is common in EM minor and occurs in as many as one third of cases, but it is
uncommon in EM major (ie, Stevens-Johnson syndrome). EM major resolves in 3-6 weeks and rarely results in scarring. EM major may have a
fulminant course that is complicated by severe secondary infection, fluid and electrolyte imbalances, and fatality in 5-15% of patients.

HIV, Pharyngitis, and Disseminated Rash

BACKGROUND
A 61-year-old woman with HIV infection and a CD4+ T-lymphocyte count of 280 cells per cubic millimeter presents with a fever, pharyngitis, and
dysphagia that have lasted 3 days. Her physical examination is remarkable for an ulcerative lesion with exudates in the left tonsillar fossa. She has no
drooling or stridor. The remaining physical findings are notable for generalized lymphadenopathy; somnolence with arousability; and a generalized rash
with macules, papules, pustules, and vesicles, which primarily appear on the face, trunk, and abdomen but not palms or soles. A CT scan of the neck is
obtained to evaluate for a retropharyngeal abscess. The CT scan reveals an abnormal, asymmetric, soft tissue thickening at the floor of the mouth and
at the base of the tongue, with epiglottic swelling and inflammation extending into the soft tissues of the neck. The parotid gland is not involved. The
white blood cell count and blood chemistry findings are normal.
What is the diagnosis?
Hint
See the stain of a skin lesion.
Answer
Disseminated herpes (varicella) zoster: Herpetic inclusions were identified in the biopsy specimen from the pharyngeal ulcer. Varicella zoster virus
(VZV) grew on the culture of vesicular fluid. Lumbar puncture findings were negative.
The human herpesvirus family includes herpes simplex virus (HSV) types 1 and 2; VZV; Epstein-Barr virus; cytomegalovirus; and human herpesviruses
6, 7, and 8. Dermatomal varicella zoster infection (ie, shingles) is usually localized and painful. Complications of herpes zoster in patients with
immunocompromise include dissemination (up to 40%), encephalitis, and visceral involvement. According to the 1993 Centers for Disease Control and
Prevention criteria, no form of VZV infection is considered an AIDS-defining illness among persons with HIV infection. HIV-infected persons, who
typically have CD4+ T-lymphocyte counts of 200-500 cells per cubic millimeter (as in this case), are more likely than immunocompetent persons to
present with reactivated VZV infection. This condition most often manifests as shingles, but several dermatomes may be involved in a substantial
proportion of patients. Recurrent episodes at the same or different sites, chronic (nonremitting) herpes zoster, and dissemination are often seen, as in
this case.
The Tzanck test in this patient revealed classic multinucleated giant cells that may be found in cases of HSV or VZV infection. Tests to differentiate the 2
infections include the use of culture, direct immunofluorescence, serology, and molecular techniques. Biopsy is warranted in patients with atypical
presentations.
Treatment for disseminated infection in patients with immunocompromise and normal renal function consists of intravenous acyclovir administered at a
dose of 10-12 mg/kg every 8 hours for 7-14 days. In older patients with normal renal function, reducing the dosage to 7.5 mg/kg every 8 hours is
recommended.

Hypoglycemia in Childhood

BACKGROUND
A 14-month-old girl is referred to a pediatric endocrinologist after she had 5 episodes of staring, limpness, and unresponsiveness beginning at 10
months of age. The episodes lasted 5-30 minutes. The first episode occurred when the child was hospitalized for a rotaviral infection. The other 4
occurred after several hours of fasting, though she was otherwise healthy. The child's blood glucose levels were measured during 2 of the episodes and
found to be 43 and 47 mg/dL. A neurologist evaluated the girl and obtained an EEG, a head MRI, a serum ammonia measurement, a lactic acid
concentration, an acyl carnitine profile, qualitative amino acid levels, and urine metabolic values. The results were all normal.
The child's past medical history is otherwise unremarkable. She was born at term with a weight of 3.5 kg and length of 50 cm (both in the 50th
percentile). Her only surgery was for the placement of tympanostomy tubes to treat recurrent otitis media. She began crawling at 9 months and walking
at 14 months.
The physical examination reveals a proportionate girl with a weight of 8.5 kg (10th percentile) and a length of 68.5 cm (<5th percentile). Further
evaluation with the following tests produced normal results: morning cortisol level, insulin level, thyroxine (T4) level, thyroid-stimulating hormone (TSH)
level, and oral glucose tolerance test. Her insulinlike growth factor-1 (IGF-1) level is undetectable.
What is the diagnosis?
Hint
Growth delay.
Answer
Growth hormone deficiency: The child has classic symptoms of growth hormone deficiency in infancy. Growth hormone is one of the counterregulatory
hormones, in addition to cortisol, glucagon, and epinephrine. Hypoglycemia in the setting of panhypopituitarism or combined growth hormone and
adrenocorticotropic hormone (ACTH) deficiencies tends be more severe. However, hypoglycemia associated with isolated growth hormone deficiency
can be significant, especially during periods of illness or fasting.
The clue in this patient's case was the deceleration in growth during her first year of life. Delays in the development of gross or fine motor skills can also
be associated with this condition. The low IGF-1 level supports the diagnosis of growth hormone deficiency. This child subsequently underwent 2 growth
hormone provocative tests, one with L-dopa and the other with clonidine. Both tests yielded growth hormone values far less than 10 ng/mL, which is
considered diagnostic.
The patient's response to growth hormone therapy has been excellent, with total resolution of hypoglycemic symptoms and marked acceleration in
linear growth. The patient's most recent growth parameters at age 4.5 years were a height of 108.7 cm (75th-90th percentile) and a weight of 18.7 kg
(75th percentile). See the infant and childhood growth curves depicting the patient's response.

Irritation and Blurred Vision in the Right Eye

BACKGROUND
A 41-year-old man visits his primary care physician because of a mild irritation in his right eye that has lasted 2 days. He also has blurred vision in that
eye. The results of fluorescein staining are positive. What is the diagnosis?
Hint
The cornea shows a branched staining area with fuzzy borders.
Answer
Herpes simplex keratitis of the right eye: Infection of the cornea by herpes simplex virus (HSV) type I typically causes corneal epithelial defects. Staining
with fluorescein or rose bengal dye causes the defects to appear as dendritic figures. Patients with HSV keratitis have unilateral ocular redness and
variable discomfort. If the more central portion of the cornea is involved, patients have blurry vision. Treatment includes the administration of antiviral
eye drops and careful observation. The use of corticosteroid eye drops is contraindicated in active epithelial herpes simplex. Recurrence after long
periods of dormancy is common, but these episodes may be reduced with the long-term administration of systemic antiherpetic agents.

Irritation and Tearing in the Right Eye

BACKGROUND
A 72-year-old man visits his primary care physician because of an intermittent worsening irritation in his right eye that has lasted approximately 4 weeks.
He has a small amount of associated tearing without discharge. His visual acuity is good. What is the diagnosis?
Hint
The patient states that his ocular irritation is temporarily relieved when he rests his right cheek on his hand.
Answer
Entropion of the right lower eyelid: Entropion is an abnormality in the position of the lower eyelid that causes it to turn inward. The turning in of the eyelid
margin and its eyelashes causes irritation of the lower conjunctiva and cornea. In this older patient, the entropion is involutional; it is caused by an
imbalance between the eyelid muscles that maintain and stabilize the normal lid position. The patient experiences immediate relief when his lower eyelid
is pulled downward because the position of his eyelashes is corrected and they are moved away from his eye. When this correction occurs, the
examination findings are normal. When he closes his eyes forcibly, his eyelid turns inward again, and a foreign body sensation results. Although taping
the lower eyelid to the cheek temporarily relieves the symptoms, only surgical management of involutional entropion provides permanent relief.

Lumps on the Legs of a Woman

BACKGROUND
A 26-year-old woman presents with a 3-week history of a tender, red, progressive rash on her legs. She has a history of exposure to tuberculosis, which
currently is not active. The patient started taking a new oral contraceptive 4 weeks before this presentation. She now is receiving a second course of
antibiotics for suspected cellulitis, but the rash has not improved. She is allergic to penicillin. What is the diagnosis and treatment?
Hint
The vital signs of this patient are within the normal ranges. She has 5 tender deep-seated nodules on her legs. No signs of trauma, insect bites, or track
marks are present. The remaining findings of her physical examination are normal.
Answer
Erythema nodosum: Findings from the CBC; sodium, potassium, chloride, carbon dioxide/bicarbonate, BUN, creatinine, and glucose (chem 7) tests;
chest radiography; throat culture; and usual laboratory studies were not consistent with tuberculosis, sarcoidosis, streptococcal infection, or yersiniosis.
The urine pregnancy test results were negative. A dermatologist was contacted, and the patient was instructed to stop taking her oral contraceptives,
elevate her legs, and begin therapy with nonsteroidal anti-inflammatory drugs (NSAIDS). She followed up with her dermatologist and private medical
doctor (PMD) several days later. The sex and age of the patient as well as the location and tenderness of the lesions are all typical of erythema
nodosum. Predisposing factors for erythema nodosum include bacterial and fungal infections, use of drugs (eg, oral contraceptive pills, sulfonamides),
pregnancy, sarcoidosis, ulcerative colitis, and idiopathic causes.

Nail Changes After Cast Immobilization

BACKGROUND
This 30-year-old man fractured his right wrist and hand in a car accident. The hand and wrist were immobilized with a plaster cast for 45 days, during
which time the patient complained of moderate paresthesia and pain.
Ten days after the cast was removed, the patient presented to the hospital with painful periungual inflammation on the fourth finger on his right hand.
Physical examination revealed proximal onychomadesis (ie, nail detachment) and a bleeding vascular nodule protruding from the proximal nail fold. The
adjacent third finger showed a Beau line.
The nail changes resolved spontaneously in 4 weeks. What is the diagnosis?
Hint
The patient denies any trauma after the cast was removed.
Detachment of the proximal nail was preceded by temporary arrest in growth of the nail, though the nail matrix was not directly damaged.
Pathologic examination of the vascular nodule reveals features of pyogenic granuloma.
Nerve conduction and electromyographic results are normal.
Answer
Paronychia and periungual pyogenic granuloma after cast immobilization: The history of pain and paresthesia of the hand during the period of
immobilization suggests that the cast might have been too tight, causing mild peripheral nerve injury. However, the mechanism by which mild injury
produces a pyogenic granuloma is unknown. This lesion is usually caused by minor wounds in the proximal nail fold.
Clinically, pyogenic granuloma may mimic nail tumors, including subungual squamous carcinoma and melanoma. About 20% of cases of nail melanoma
are amelanotic.
Cast immobilization should be included among the possible causes of periungual pyogenic granuloma. When a patient complains of pain and
paresthesia of the hand during or after immobilization, the possibility of nail damage due to an improperly applied cast should be considered.
For more information about paronychia and pyogenic granuloma, see the eMedicine articles Paronychia and Pyogenic Granuloma (Lobular Capillary
Hemangioma).

Newborn With Hypoglycemia and a Micropenis

BACKGROUND
This full-term male newborn is noted to have intractable hypoglycemia. His blood glucose level is 20-30 mg/dL. The child's physical examination findings
are normal except for a small penis. The newborn's blood glucose level can be corrected with intravenous (IV) glucose, but he becomes hypoglycemic
when the glucose infusion is stopped. What is the diagnosis?
Hint
What counterregulatory hormones counteract hypoglycemia?
Answer
Hypopituitarism, with deficiencies of adrenocorticotropic hormone (ACTH), thyroid-stimulating hormone (TSH), luteinizing hormone (LH), folliclestimulating hormone (FSH), and growth hormone (GH): The term micropenis is used when the stretched length of the penis is more than 2.5 standard
deviations (SDs) less than the mean for the person's age. For a newborn, the criterion is less than 2 cm. Micropenis can result from various causes: (1)
inadequate androgen production secondary to inadequate gonadotropin secretion, (2) inadequate androgen production because of primary gonadal
failure (including deficiency of an enzyme necessary for testosterone production), (3) partial androgen insensitivity, or (4) an idiopathic cause. Profound
hypoglycemia may result from an absence of glucose counterregulatory hormones, including cortisol, GH, epinephrine, and glucagon.
The combination of hypoglycemia with a micropenis strongly suggests pituitary deficiencies. Gonadotropin deficiencies account for the micropenis, and
ACTH and GH deficiencies account for the hypoglycemia. This child was found to have hypopituitarism with deficiencies of ACTH, TSH, LH, FSH, and
GH. Administration of small doses of testosterone resulted in penile growth, and replacement of cortisol and GH corrected the hypoglycemia.

Painful Toe Lesion

BACKGROUND
What is this painful toe lesion?
Hint
The diagnosis might be more obvious if the lesion were on the great toe.
Answer
Gout with ruptured tophus of the fourth toe: Tophi appear as raised, glistening, white papules adjacent to joints. They often occur in the feet, legs, or
forearms. Tophi are more common in elderly persons and in patients with chronic renal failure or other chronic diseases. Cyclosporine, steroids,
estrogens, and many other medications have uricosuric activity. Therapies include the short-term administration of colchicine, nonsteroidal antiinflammatory drugs (NSAIDs), and intra-articular glucocorticoids. Reference: Rosen P, ed. Emergency Medicine: Concepts and Clinical Practice. 4th ed.
Mosby-Year Book, Inc; 1998:2735-6.

Rash on a Child's Legs

BACKGROUND
One week ago, the mother of this 4-year-old girl noted small red dots on the child's legs, which the primary pediatrician diagnosed as bug bites. The
lesions became larger, and a schoolteacher confronted the mother about what she thought were bruises on the child's legs. Concerned, the mother
brought the child to a dermatologist, and biopsy was performed; however, the mother states that no diagnosis was made. Now, nearly hysterical, the
mother brings her daughter to the emergency department and complains that the rash looks worse and that the child refuses to walk. The girl is afebrile,
appears well, and she is able to ambulate. What is the diagnosis?
Hint
Children with this idiopathic syndrome often complain of abdominal pain. In some, intussusception or renal dysfunction may even develop. When
pressure is placed on the skin lesions, they are palpable but do not blanch.
Answer
Henoch-Schonlein purpura (HSP), anaphylactoid purpura: The classic syndrome of HSP consists of a purpuric rash, arthritis, gastrointestinal symptoms,
and renal involvement. Skin lesions are required for the diagnosis and often appear as small wheals or erythematous macules that progress to purpura.
Angioedema of the face, hands, feet, and perineum is also common. Two thirds of the patients have arthritis in the large joints, and more than half have
gastrointestinal symptoms. Colicky abdominal pain and vomiting are typical. Hemoccult results may be positive, or the stool can be grossly bloody. The
failure to recognize HSP can result in unnecessary laparotomy; however, intussusception or small-bowel obstruction rarely complicates HSP.
Approximately 25-50% of the patients have renal involvement. Hematuria and proteinuria are the most common findings, but patients can have
hypertension, azotemia, oliguria, or nephrotic syndrome as well. Central nervous system involvement is extremely rare. Most patients have a benign
course and require only nonsteroidal anti-inflammatory drugs (NSAIDs) for symptomatic relief. Relapses may occur. Patients with intussusception,
small-bowel obstruction, or nephrotic syndrome are given prednisone 1-2 mg/kg/d and require hospitalization. Reference: Nelson Textbook of
Pediatrics. 15th ed. Philadelphia, Pa: WB Saunders and Co; 1996.

Rash on a Woman's Back

BACKGROUND
A 41-year-old woman presents to your clinic with a sore throat, a stuffy nose, and a nonproductive cough lasting for several days. She reports subjective
fevers but no chills, shortness of breath, chest pains, or lightheadedness. On physical examination, the patient's lungs are clear to auscultation, and her
work of breathing is normal. Her tympanic membranes are translucent without erythema or fluid. She has no facial tenderness to palpation, though you
observe crusting of purulent material on the external nares. Her oral temperature is 99.4F, with a heart rate of 110 beats per minute and a blood
pressure of 125/78 mm Hg. What is the diagnosis and cause?
Hint
The patient is of Vietnamese heritage.
Answer
Coining: The patient likely has a viral upper respiratory infection, and her dermatologic condition is caused by cao gio, or coining, an ancient Southeast
Asian remedy used to treat minor ailments. Coining is performed to create a channel though which "wind," which is believed to cause the ailment, can
escape from the body.
In this remedy, hot mentholated oil is applied to the patient's back and shoulders. Then, a metal coin or spoon is rubbed repetitively, in linear fashion,
along the patient's spine, back, or posterior aspect of the ribs. This rubbing is usually not painful, but it can result in petechiae or ecchymoses. Minor
burns are the most common complication, though more serious burns requiring hospitalization and burn management can also occur. The marks can
resemble the red marks left by cigarette burns. When these findings are noted in pediatric patients, physicians should avoid interpreting them as signs of
child abuse.
In addition to coining, other common folk-healing practices include cupping and moxibustion. Cupping is the application of heated cup over the skin,
which creates a vacuum action as it cools. Practitioners of cupping believe that the patient's ailment is due to a lack of oxygen to the cells in the affected
area secondary to sluggish movement of lymph and blood. The vacuum is thought to alleviate the pathologic condition. Moxibustion is the application of
heated incense to skin.

Rash on the Face of an Infant

BACKGROUND
This male infant presented with a rash on his face. His vital signs are stable, and he otherwise appears well. What is the diagnosis? What is the
infectious organism? What serious diseases are linked to this organism? How is the disease managed?
Hint
This disease has 2 names. For extra credit, name the related 4 diseases that were discovered before this one.
Answer
Fifth disease (ie, erythema infectiosum): Erythema infectiosum is caused by parvovirus B19, which is spread by aerosolized secretions. Infection
involves a rash with 3 stages: In stage 1, the rash begins with the abrupt onset of bright erythema on the cheeks, which makes the patient appear as if
he or she has been slapped. In stage 2, this rash is followed by a maculopapular rash on the trunk and extremities. Finally, in stage 3, the rash fades
and leaves a lacelike pattern with central clearing. Parvovirus B19 is identified as a cause of hydrops fetalis. It also directly causes fetal anemia by
reducing erythropoiesis. This virus is the most common cause of aplastic crisis in children and adults with a chronic hemolytic anemia, such as sickle
cell anemia, spherocytosis, or sickle cell thalassemia. It may cause pancytopenia in patients with immunosuppression. Treatment is supportive. To
answer the extra credit question, the 4 diseases discovered before erythema infectiosum are the following: measles (ie, first disease), scarlet fever (ie,
second disease), rubella (ie, third disease), and Duke disease (ie, fourth disease). Exanthem subitum is also known as the sixth disease.

Recalcitrant Genital Disease

BACKGROUND
A 60-year-old woman presents with vaginal itching and dyspareunia that has lasted several years. Examination reveals inflammation of the vulva with
sclerosis and several erosions extending to the vagina (Image A). Prior treatment with betamethasone ointment provided some relief but caused
atrophy. What is the diagnosis?
Hint
The chronic inflammation and sclerosis cause severe symptoms.
Answer
Lichen sclerosus et atrophicus (LSA): LSA remains a disease of unknown pathogenesis that often severely affects the patient's quality of life. LSA
affects 14 people per 100,000 population per year and has a predilection for females, with up to 15% of cases involving patients in the first or second
decades of life (Powell, 1999; Hengge, 2000). Alopecia areata, vitiligo, and serologic and clinical disorders of the thyroid appear to be associated with
LSA. Experimental studies have revealed the presence of autoantibodies to the extracellular matrix (ECM) protein, which suggests an autoimmune
pathogenesis (Oyama, 2003).
The clinical presentation includes symptoms affecting the anogenital area (vulva, penis, anus). Although any skin site, including the oral mucosa, may
be affected (Jensen, 2002), LSA most frequently occurs on the vulva, where it may be asymptomatic or cause intractable itching and soreness (Powell,
1999; Hengge, 2000). Typical clinical findings are telangiectasia, purpura, erosions, variable degrees of sclerosis, and polygonal papules and plaques
with atrophic fragile skin. With time, progression to destructive sclerosis commonly leads to narrowing of the anogenital orifices. Extragenital sites
affected include the thighs; the submammary region; the neck; the shoulders; and, sometimes, the oral mucosa. Differential diagnoses include erosive
lichen planus, scaring pemphigoid of mucous membranes, lichen simplex, morphea, and balanitis obliterans xerotica.
The inflammatory infiltrate of LSA has recently been shown to contain CD8/CD57-positive epidermotropic lymphocytes, which commonly serve as
markers for a chronic excessive antigen deposition (Carlson, 2000). In addition, intraepidermal major histocompatibility complex (MHC) class IIpositive
cells are also increased (Farrell, 1999). Furthermore, levels of interleukin (IL)4 and transforming growth factorb are elevated in LSA (Scrimin, 2000).
Asymptomatic extragenital LSA generally requires no treatment other than that to control pruritus. Various attempts to treat genital LSA have been
undertaken. The only treatment proven to be effective seems to be long-term therapy with topical steroids of decreasing potency, as evidenced in
nonrandomized controlled studies (Dahlmann-Ghozlan, 1999; Sinha, 1999). Randomized studies have not shown a significant effect of topical
testosterone or estrogen, vitamins, cyclosporin, retinoids, or antibiotics.
Photodynamic therapy has been used (Hillemanns, 1999), but this treatment can be painful. Destructive surgical therapy should be used only if
squamous cell cancer is detected. In general, the lifetime risk of squamous cell cancer as a complication of long-standing LSA is estimated to be 4-6%
(Powell, 1999). LSA tends to recur, especially after circumcision in males, and may lead to radical surgery.
Given the prominent symptoms and limited therapeutic options of LSA, novel treatments are highly desirable. The novel topical immunomodulator (TIM)
tacrolimus was evaluated as a therapeutic alternative for this difficult-to-treat patient. In this case, tacrolimus 0.1% ointment was applied two times per
day. All of the patient's symptoms and signs, except for sclerosis, subsided after 6 weeks of treatment (Image B). She did not experience any worsening
of burning or itching while receiving treatment.
Topical tacrolimus belongs to the new class of topical TIMs, which also includes cyclosporin and pimecrolimus. It has been approved for the treatment of
atopic dermatitis (Reitamo, 2001). Tacrolimus is a lipophilic compound that inhibits the phosphatase calcineurin, blocking the transcription of
proinflammatory cytokines such as IL-2 and interferon-g (Assmann, 2001). Calcineurin inhibitors reduce antigen processing and T-cell activation. They
also affect other cell types involved in itching and inflammation, such as mast cells and eosinophilic and basophilic granulocytes (via inhibition of IL-3,
IL-8, IL-13, granulocyte/macrophage colony-stimulating factor [GM-CSF], histamine, and tryptase) (Scrimin, 2000). FceR1 receptor expression is also
reduced on epidermal antigen-presenting cells (Wollenberg, 2001).
Although many effects of topical tacrolimus parallel those of corticosteroids, adverse effects such as skin atrophy and telangiectasias do not occur with
TIMs. This drug has been used for off-label indications such as the treatment of eczema of the eyelids (Mayer, 2001), perioral dermatitis and rosacea
(Goldman, 2001), lichen planus (Kaliakatsou, 2002), pyoderma gangrenosum (Reich, 1998), and vitiligo (Grimes, 2002; Lepe, 2003).
Patient-applied tacrolimus, the first TIM for chronic inflammatory diseases, bears potential in the treatment of LSA. A trial has been initiated to confirm its
effect on this condition.
For more information on LSA, see the eMedicine articles Lichen Sclerosus et Atrophicus and Balanitis Xerotica Obliterans (within the Dermatology
specialty).

Red and White Baby

BACKGROUND
The parents of this 3-day-old baby arrive in the emergency department stating that the newborn has been acting well since being discharged from the
hospital yesterday. She is breastfeeding every 2-3 hours, moving her bowels multiple times daily, wetting her diapers after each feeding, and consoling
appropriately. However, the parents report that, when they placed the baby on her side earlier today, she turned different colors?the side down was red,
and the side up was white. On examination, the baby appears normal, but when laid on her right side, her skin changes colors, as demonstrated in the
photograph. What is this phenomenon called? What causes the color change to occur?
Hint
This is a benign condition of neonates.
Answer
Harlequin color change: The harlequin color change results from neurovascular immaturity of the blood vessels in newborns. The sharply demarcated
line between the inferior erythema and the superior pallor occurs in 10% of healthy newborns when they are placed on their sides. Premature infants are
more prone to this harlequin color change. The incidence peaks between the second and fifth days of life.
Episodes of discoloration persist 30-120 seconds, and crying or increased movement or activity decreases the duration of the color change. The
condition is entirely benign and unrelated to other medical diagnoses. Families should be counseled that the color change is not a cause for worry.
Harlequin color change should not be confused with the diagnosis of harlequin fetus, a severe, restrictive skin disorder of neonates.

Red Eyelids

BACKGROUND
An 18-year-old woman presents to the emergency department with red eyelids. She states that the swelling and itching in the right eyelid began 3
months ago and that it was diagnosed as a sty. Initially, a physician gave her mupirocin ointment, but the rash and swelling worsened and now affects
both eyes. The patient denies having any eye pain, visual changes, or drainage. She is taking Paxil for depression and has a 17-month-old daughter
who was born by means of cesarean delivery. According to the patient's family history, both her uncle and cousin had systemic lupus erythematous. Her
vital signs are as follows: blood pressure, 124 mm Hg systolic, 82 mm Hg diastolic; heart rate, 97 beats per minute; respiratory rate, 14 respirations per
minute; temperature, 97.6F.
Hint
The patient also complains of a 3-month history of generalized fatigue, myalgias, and muscle weakness. Her endurance has decreased substantially.
For instance, when she goes to the grocery store, she must lean on a cart. The patient also cannot rise from a chair without using her hands. She
associates the onset of her myalgias and weakness to a minor motor vehicle accident. She was given indomethacin a couple of months ago, but her
symptoms have only become worse. Her serum creatine kinase (CK) level, which was obtained in the emergency department, is 1355 U/L (normal
range, 40-200 U/L).
Answer
Dermatomyositis: The heliotrope eyelids are described as a red-purple or violaceous, edematous, macular rash. This finding is characteristic of
dermatomyositis, an idiopathic connective-tissue disease that is characterized by inflammation of the muscles and skin. Pathognomic manifestations
include Gottron papules, which are violaceous papules overlying the dorsal surface of the elbows or knees or the interphalangeal or
metacarpophalangeal joints of the fingers. Patients may present with proximal muscle weakness (as this patient did), dysphonia, dysphagia, respiratory
muscle weakness, visual changes, or even abdominal pain. Cardiac complications include cardiomyopathy and cardiac conduction defects. Pulmonary
complications include aspiration pneumonia, interstitial pneumonitis, and fibrosis. Ophthalmic complications include iritis, nystagmus, cotton-wool spots,
conjunctival edema, and optic atrophy. Other complications include an increased risk of malignancy, especially in adults with ovarian and gastric
cancers, and lymphoma. Polymyositis is an associated inflammatory myopathy without the cutaneous findings. Skin biopsy findings are not specific for
dermatomyositis, but they may help in excluding other skin conditions. After elevated serum CK and aldolase levels were obtained in this patient, the
diagnosis of dermatomyositis was confirmed by means of muscle biopsy. The routine evaluation also includes chest radiography; electrocardiography
(ECG); urinalysis (to assess for myoglobin); evaluation of the complete blood count (CBC); and determinations of serum lactate dehydrogenase (LDH),
alanine aminotransferase (ALT), aspartate aminotransferase (AST), and antinuclear antibody (ANA) levels. A stool guaiac test, mammography, and
vaginal ultrasonography should be performed for cancer screening. Her elevated CK level, muscle weakness, and eyelid redness improved with daily
administration of 20 mg of prednisone, which was begun during her visit to the emergency department. She followed up with a rheumatologist. Source:
Koler RA, Montemarano A. Dermatomyositis. Am Fam Phys. 2001;64:1565-1572.

Red Eyes

BACKGROUND
A 9-year-old girl presents to her primary care physician with red eyes. Her father says that her eyes have appeared bloody. The girl has no history of
trauma. The patient reports a somewhat gritty sensation in her eyes, but she has otherwise been feeling well. The examination reveals bilateral
extensive subconjunctival hemorrhages. What is the diagnosis?
Hint
A number of children in the patient's class at school have been absent because of "pinkeye."
Answer
Bilateral hemorrhagic conjunctivitis: In children, an external infection with Picornaviridae viral species, specifically Coxsackievirus or Enterovirus, is
characterized by breakdown of the capillary walls within the conjunctivae, with resulting subconjunctival hemorrhage. This condition is equivalent to
purpura of the skin. Haemophilus influenza infection may cause a similar clinical picture, but this bacterial infection usually causes significant discharge,
whereas the viral infection does not.
These viruses are highly contagious. The disease is self-limited, and treatment is symptomatic. If H influenza infection is suspected, obtain a culture and
wait for the results before instituting treatment with appropriate topical antibiotics.

Red, Tender, and Swollen Eyelid

BACKGROUND
A mother brings her 12-year-old boy to the hospital because of pain, redness, and increased swelling in the left lower eyelid for the past 2 days. The boy
denies any injury to his face but admits to exploring a junkyard with his friend last weekend. The boy has no history of previous eye problems, systemic
injuries, or allergies.
What is the diagnosis?
Hint
The mother points out a small scab on her son's lower eyelid.
Answer
Examination reveals moderate pain and distress. The boy's left lower eyelid is swollen and tender to palpation, and a small, 5- to 6-mm scab is adjacent
to the most tender swollen area.
Radiographs depict a 6-mm foreign body retained in the anterior aspect of the left inferior orbit. The object is made of brass and cylindrical, cupped, and
smooth. Examination of the globe and extraocular muscles does not reveal any injuries.
The object is removed by means of sharp dissection, and the small entry wound is closed with cosmetic closure techniques. Cultures of the wound are
negative, but the patient is given a broad-spectrum antibiotic for a few days.
Follow-up examinations show no sequelae. On further questioning, the boy admits to shooting BBs at cartridges, tin cans, and bottles found in the
junkyard.
For more information on foreign bodies of the eye, see the eMedicine article Foreign Body, Intraocular.

Severe Chronic Fatigue in a 54-Year-Old Woman

BACKGROUND
A 54-year-old woman presents with chronic fatigue that has lasted many years. She has recently moved to the area from another state. Her history
includes a hysterectomy 19 years ago for dysfunctional uterine bleeding and hypophysectomy and irradiation 14 years ago for a pituitary tumor. Her
current medications include glucocorticoids (prednisone, 7.5 mg/d); levothyroxine, 100 mcg/d; and conjugated estrogen, 0.625 mg/d. She has also been
taking 1500 mg of calcium carbonate and 600 IU of vitamin D every day. The woman is a smoker and has failed several attempts to quit. No other
significant medical history is recorded. She had read an article about growth hormone (GH) therapy and is seeking advice. The physical examination
findings were remarkable for a round (Cushingoid) face and elevated systolic blood pressure (156 mm Hg systolic, 82 mm Hg diastolic). The woman is 5
ft 5 in tall and weighs 142 lb. She denies any past history of hypertension, dyslipidemia, cardiovascular disease, or diabetes mellitus.
After discussing the possible benefits and risks of GH therapy in adults, recombinant human growth hormone (rhGH) treatment is initiated at a low dose
of 6 mcg/kg/d (see image for chemical structure). After 3 weeks of therapy, the patient complains of arthralgias but notices marked improvement in her
strength and energy. Because of the arthralgias, GH therapy is stopped. Her arthralgias resolve, but her fatigue returns. The patient requests to start
rhGH therapy again. After restarting rhGH treatment, the patient does not experience any arthralgias, and she notices marked improvement in her
overall well-being.
Over the last 2 years, the patient's metabolic status has not changed, and her insulinlike growth factor-1 (IGF-1) level is at the 50th percentile for her
age. She remains energetic and refuses to discontinue GH therapy. Her current dose is 12 mcg/kg/d. No objective changes are correlated with the
patient's subjective feeling of well-being. Bone-density dual-energy x-ray absorptiometry (DEXA) does not show any evidence of osteoporosis. What is
the diagnosis?
Hint
The patient underwent a significant neurosurgical procedure 14 years before presentation.
Answer
Panhypopituitarism following hypophysectomy and irradiation ? Growth hormone deficiency: This patient has acquired hypopituitarism due to
hypophysectomy and cranial irradiation, which were used to treat a pituitary tumor that was diagnosed 14 years before this presentation. Therapy with
glucocorticoids, thyroid hormone, and estrogens was adequate. However, after surgery and irradiation, her fatigue was unrelenting. When the patient
was given rhGH therapy, her fatigue markedly improved. She had arthralgias when treatment was first started. However, she did not report any
arthralgias when therapy was reinstituted.
This patient did not require any dynamic testing because her GH deficiency was due to surgery and irradiation. Her IGF-1 level was reduced at 96
ng/mL. However, an IGF-1 measurement is not needed for diagnosis because several factors, including nutrition, exercise, and steroid use, can affect
the result. In this patient, the IGF-1 level was measured to avoid overreplacement; her current IGF-1 level was at the 50th percentile for age.
In August 1996, the Food and Drug Administration (FDA) approved the use of GH therapy in adults with GH deficiency. Several studies reported that
individuals with GH deficiency have an increased risk of cardiovascular mortality, osteopenia and osteoporosis, fractures, changes in body composition,
reduced exercise tolerance, and decreased muscle strength. Improvements in many of these parameters are reported after replacement therapy.
Improvements in quality of life have been described in most studies.
Some investigators question the robustness of the evidence for most changes other than improvements in bone mass. A discussion of this topic can be
found in the August 6, 2002, issue of Annals of Internal Medicine. Some argue that most of the benefits ascribed to GH therapy may be due to a
placebo effect. On the other hand, patient testimonials (including this case) may suggest that GH therapy should be offered to patients who have
participated in an informed discussion and understand the costs involved and concerns regarding long-term safety (which remains to be established).
Groups studying IGF-1 question whether patients taking pharmacologic doses of GH might be at a higher risk for malignancies (eg, lymphoma or
uterine, breast, or prostate cancer). Clinical research on the use of GH in adults continues.

Shoulder Pain

BACKGROUND
This young woman tripped down a flight of stairs and fell, landing on her left shoulder. She complains of localized pain and swelling over the
sternoclavicular joint. Her extremities are neurovascularly intact. What tests should be performed? What is the diagnosis?
Hint
She is breathing comfortably but complains of pain on clavicular or arm motion.
Answer
Anterior sternoclavicular joint dislocation: Dislocation of the sternoclavicular joint is rare and primarily a clinical diagnosis. Anteroposterior and cephalic
tilt radiography is indicated. The cephalic tilt view is obtained by tilting the x-ray beam 40? in the cephalic direction. However, CT scanning is becoming
the preferred imaging modality for the evaluation of sternoclavicular injuries. In this patient, the radiographs showed an anterior subluxation but no
fracture. Oral pain medication was given, a sling was applied, and an orthopedic surgeon was consulted to reduce the injury with the patient under
conscious sedation. Acute posterior sternoclavicular joint dislocations are true orthopedic emergencies. The medial end of the clavicle becomes
posteriorly displaced behind the sternum, placing pressure on the underlying trachea that can cause airway obstruction. The patient usually has severe
pain and complains of pressure on his or her throat.

Simultaneous Onset of Diabetes and Rash

BACKGROUND
A previously healthy 70-year-old woman presented to her primary care physician with the onset of diabetes mellitus, unintentional weight loss, anemia,
and diarrhea. Shortly thereafter, glossitis developed, along with a widespread pruritic rash that was recalcitrant to treatment with systemic and topical
steroids. At the time of her referral to a dermatologist, her trunk and extremities were covered with gyrate, dusky tan-pink and bright pink-red plaques
with exfoliative, hemorrhagic, and varnishlike scales; these did not affect the perianal area, hands, or feet.
Skin biopsy revealed broad zones of confluent parakeratosis, an edematous dermis with a mixed infiltrate of lymphocytes, histiocytes, and occasional
eosinophils. Immunofluorescence findings were negative. Subsequent studies indicate a markedly elevated fasting glucagon level and low serum amino
acid and essential fatty acid levels.
What are the diagnosis and most appropriate treatment?
Hint
CT scans revealed 2 masses in the pancreas (not shown).
Answer
Necrolytic migratory erythema: This patient^s rash is consistent with necrolytic migratory erythema (NME), as part of the glucagonoma syndrome. Initial
treatment with octreotide 200 mcg 3 times a day resulted in rapid symptomatic relief and resolution of her rash. She then underwent total
pancreatectomy with the removal of regional metastases.
Glucagonoma syndrome consists of a glucagon-secreting tumor, diabetes mellitus, NME, anemia, hypoaminoaciduria, and weight loss. NME is often the
most specific and readily diagnosable component of this syndrome and has been shown to be the presenting complaint in as many as two thirds of
cases. The typical rash of NME appears as erythematous patches and plaques that develop flaccid vesicles or bullae, which later break resulting in
denudation and crust formation. The differential diagnosis includes eczema, psoriasis, impetigo, acrodermatitis enteropathica (an eruption secondary to
zinc deficiency), subcorneal pustular dermatosis, and autoimmune blistering diseases. The rash does not typically mirror glucagon levels and has a
waxing-and-waning course with cycles lasting 7-14 days.
The key to the diagnosis in this patient was the simultaneous onset of rash and diabetes mellitus. NME most commonly occurs during the fifth or sixth
decades of life. Glucagon levels cannot be checked in every patient in this age group who has a new onset of diabetes mellitus; however, the abrupt
occurrence of recalcitrant new rash and diabetes mellitus should prompt testing of fasting glucagon levels. Early diagnosis of glucagonoma syndrome is
key to survival because metastatic disease is often present when the diagnosis is made. NME with normal glucagon levels has also been reported in the
setting of cystic fibrosis, pancreatitis, and celiac sprue.
For more information on glucagonoma syndrome, see the eMedicine articles Glucagonoma Syndrome and Glucagonoma.

Sore Throat and Ear Pain

BACKGROUND
A 29-year-old man presents to the hospital with a sore throat and left ear pain that have lasted 5 days. He has a history of bipolar disorder and is taking
lithium. He has had some dysphagia, fever to a maximum temperature of 102?F (38.9?C), and no ill contacts. He was treated at a different hospital and
given a 5-day course of antibiotics, which he finished on the day of presentation. He has not been eating much because the pain is becoming worse. His
current temperature is 99.7?F (37.6?C), and his other vital signs are stable. On examination, cervical lymphadenopathy, a marked left facial droop, and
vesicles in his left ear are noted. The tympanic membrane is dull but not infested with vesicles. The patient has erythema and blisters on the left side of
the tongue. What is the diagnosis, and how is it treated?
Hint
The patient also complains that his left eye is dry and that vision in that eye is blurry. Physical examination reveals the following: pupils equal, round,
and reactive to light (PERRL); extraocular movements intact (EOMI); no diplopia; and no obvious conjunctivitis or other external eye problems. The
patient has a severe corneal abrasion from not being able to completely close his left eye. No dendrites are seen on fluorescein examination. Facial
sensation is intact. Hint: Consider a possible syndrome.
Answer
Ramsay Hunt syndrome: This syndrome consists of facial palsy that is sometimes accompanied by loss of taste in the anterior part of the tongue,
tinnitus, hearing loss, and vertigo. The syndrome is caused by a herpes zoster infection of the geniculate ganglion. This patient received intravenous
doses of acyclovir, Solumedrol, and morphine. A patch was placed on his eye, and he was given tobramycin ophthalmic solution and artificial tears. The
patient was discharged with oral acyclovir and prednisone (to be tapered). He is scheduled for follow-up with a neurologist and an ophthalmologist.
Reference: Braunwald E, Fauci A, Hauser S, et al. Harrison Principles of Internal Medicine. 15th ed. McGraw-Hill Professional; 2001:2350.

Struck in the Eye

BACKGROUND
An 18-year-old man visits his primary care physician 2 days after being struck in the right eye with a fist during an altercation. Initially, he could not see
out of the eye and had a moderate amount of pain; this was perhaps decreased to some extent by his blood alcohol level at the time of the incident. By
mid morning of the following day (yesterday), the patient's vision had improved significantly, and the pain had disappeared. Examination with penlight
reveals a sluggish pupil and a planoconvex red area seen in the photograph. What is the diagnosis?
Hint
Visual acuity in the right eye is 20/20, but the patient states that his vision was worse when he woke up this morning.
Answer
Layered traumatic hyphema of the right eye: Hyphema, or blood in the anterior chamber, is usually the result of external trauma or surgery. It can also
occur in the absence of trauma, eg, in patients with iris neovascularization after central retinal vein occlusion or related to diabetes mellitus. In rare
cases, hyphema may be a sign of an iris tumor.
In this case, the blunt blow to the eye ruptured a blood vessel of the iris, causing a small amount of blood to leak into the anterior chamber. Initially, the
blood was dispersed within the aqueous humor, but over the next few hours, the blood settled into layers within the inferior portion of the anterior
chamber. Typically, when the person is recumbent (eg, during sleep), any blood remaining in the chamber again mixes with the aqueous humor,
causing blurred vision at the time of wakening.
Traumatic hyphema may be associated with many other injuries to the eye, depending on the nature and severity of the blow. Injury to the iris
musculature that causes traumatic iridoplegia may be present, as in this case. This injury manifests as a slightly dilated and sluggish pupil. In the
absence of rebleeding, most hyphemas spontaneously clear within a few days.
Treatment includes avoidance of any physical activity that might cause rebleeding; the application of topical corticosteroid eyedrops to reduce
inflammation of the iris and ciliary body; and, occasionally, the use of cycloplegic drops to relieve ciliary spasm or prevent adhesions from forming
between the iris and the anterior lens. Bed rest is not necessary in uncomplicated cases.
All patients with traumatic hyphema should undergo ophthalmologic examination after the hyphema has cleared to rule out peripheral retinal tears and
damage to the anterior chamber angle.

Struck in the Eye

BACKGROUND
A 36-year-old man with presents to the emergency department 15 minutes after being struck in his right eye (OD) while playing basketball. He has loss
of vision and pain in his right eye but not in his left eye (OS). He no other facial or head trauma and no other complaints.
His visual acuity is recorded as OS 20/20 and OD 20/400, without pinhole improvement. The left pupil is 2 mm and reactive, whereas the right pupil is 6
mm and minimally reactive. The patient has red discoloration over 30% of the inferior aspect of the right anterior chamber (top image). No afferent
pupillary defect is observed, though the cornea of the affected eye has a hazy appearance. Intraocular pressures (IOPs) measured on tonometry are OS
12 and OD 42 mm Hg. On fluorescein examination, no corneal abrasions are observed and the slitlamp examination reveals cells in the OD anterior
chamber. The fundi are within normal limits.
What is the diagnosis and pharmacologic treatment? Why is checking for an afferent papillary defect (APD) important?
Hint
The red discoloration of the anterior chamber and the corneal haziness are abnormal.
Answer
Hyphema with traumatic iritis and increased IOP: The patient has a hyphema with traumatic iritis and an acute increase in IOP. Postinjury accumulation
of blood in the anterior chamber can be a sign of major intraocular trauma with associated damage to vascular and other intraocular tissues. Blunt
trauma to the globe results in tears to the ciliary body, iris, and other anterior segment structures. These tears cause shearing of blood vessels,
including those that make up the major arterial circle of the anterior segment and are responsible for hyphema formation. The potential for secondary
hemorrhage and persistent elevations in IOP can result in poor final visual result.
Symptoms of hyphema include pain and photophobia. The patient's visual acuity can be affected as a result of obstructing cells and blood. Initially, the
IOP can markedly rise, as in this case, because of the disruption of the normal egress of aqueous humor into the trabecular meshwork from clogging by
red blood cells and their byproducts or from direct trauma to the meshwork itself from the initial trauma.
Regarding the management of acute traumatic hyphema, first assess for concomitant injuries (eg, globe rupture, intraocular foreign body) that require
emergency consultation with an ophthalmologist. If no emergency associated injuries exist, the goal of acute management is minimizing the risk of
rebleeding and decreasing any pathologic increase in IOP. Rebleeding can occur with any activity that places additional shear forces on the affected
blood vessels. Patients should avoid activities such as reading, tasks requiring excessive eye movements, or moving between different lighting
conditions, which may cause papillary play (ie, the normal constriction and dilation of the iris in response to light). A long-acting cycloplegic agent, such
as atropine or cyclopentolate (ophthalmic) can eliminate accommodation and dilate the pupil to desired effect. Topical steroids can help with associated
iritis.
IOP above 24-30 mm Hg should be treated. The hyphema should be allowed to layer. Pharmacologic therapy with a topical beta-blocker (which
decreases the production of aqueous humor) should be initiated. Use of carbonic anhydrase inhibitor agent, which rapidly decreases the production of
aqueous humor, should be considered in patients who do not have sickle cell disease, as should an osmotic diuretic agent, such as intravenous
mannitol or oral glycerol. Activities that increase IOP (eg, coughing, vomiting) should be avoided.
This patient was treated by ordering bed rest, elevating the head of the bed, and administering a topical beta-blocker and a carbonic anhydrase
inhibitor. The hyphema decreased in volume and became layered by gravity (bottom image). In addition, his posttherapeutic IOP was 19 mm Hg, which
made surgical washout of the anterior chamber unnecessary. A hard eye patch was placed, and follow-up with an ophthalmologist was arranged for the
next morning.
Hyphema or glaucoma does not usually result in an APD unless the optic nerve is damaged as a result of persistent or marked elevations in IOP.
Reducing this pressure on an emergency basis can help preserve the vision of patients with a hyphema and associated elevations in IOP.
For more information about hyphema, see the eMedicine articles Hyphema and Glaucoma, Hyphema (within the Ophthalmology specialty) and Eye
Injuries (within the Consumer Health specialty).

Sudden Ankle Pain During a Basketball Game

BACKGROUND
A middle-aged man presents to the emergency department after hearing a pop while playing basketball. What classic sign is present? Name the injury
and explain its management.
Hint
Look where the finger is pointing. Patients with this injury may delay seeking medical care because their initial symptoms may be limited to weakness
that occurs with a push-off movement. Pain, edema, and ecchymosis may follow.
Answer
Achilles tendon rupture: This is caused by sudden dorsiflexion when a person runs with his or her knee extended. A direct blow to the Achilles tendon
while the gastrocnemius and soleus muscles are contracted also may result in rupture. Ruptures usually occur 2-6 cm proximal to the insertion point
because the poor blood supply is poorest in this area, with resultant degeneration. The photo illustrates palpation of the defect with the patient prone
and the affected foot hanging over the edge of the bed. The result of the Thompson test is positive, that is, no plantar flexion when the gastrocnemius
muscle is squeezed. Rest, ice, compression, and elevation (RICE) should be the initial therapy in the emergency department. Partial tears are managed
with posterior splints and crutches and with no weight bearing. Surgical repair has a lower re-rupture rate (2%) than nonsurgical casting (10-25%), but
both require 6 weeks of immobilization, followed by 5 weeks of protected weight bearing and 6 months of rehabilitation. Reference: Tintinalli JE, Krome
RL, eds. In: Emergency Medicine: A Comprehensive Study Guide. 4th ed. 1995:1302.

Swelling and Discharge in the Left Eye

BACKGROUND
A 7-year-old boy presents to his primary care physician with a swollen left eye. Three days ago, what was thought to be a pimple developed below his
left eye, adjacent to his nose. Two days ago, the left eyelids appeared erythematous and swollen. Yesterday, the swelling became worse, and the
patient's mother noted discharge from his left eye. In addition, he had a low-grade fever and complained of having a headache and body aches. What is
the diagnosis?
Hint
The patient's father is in the US Army Special Forces and is being deployed to Iraq next week.
Answer
Vaccinia of the eyelid caused by inadvertent inoculation: Vaccinia virus is the live virus used in the smallpox vaccine. It is a pox-type virus related to
smallpox. When given to humans as a vaccine, it helps the body develop immunity to smallpox. Because the virus in the vaccine is live, it can be
inadvertently transmitted from the recently vaccinated person to other people. Bandages, clothing, and hands that touch the vaccination site before it
heals can become contaminated and transmit the virus to another person. Care of the vaccination site must be taken to prevent spreading of the
vaccinia virus.
Treatment of this patient involves consultation with an ophthalmologist to assess the eye and the administration of vaccinia immunoglobulin (VIG)
together with systemic antibiotics to prevent and treat any secondary bacterial infection. The differential diagnosis includes primary herpes simplex
infection of the eyelid; however, in this case, the recent history of a family member being vaccinated against smallpox confirms the diagnosis of vaccinia.

Swollen Catheter Site

BACKGROUND
This 74-year-old man presents to the hospital with chills and an elevated temperature of 102.2?F (39?C). He has a history of acute myeloid leukemia
(AML). His vital signs are stable. A Hickman tunneled indwelling catheter had been pulled from the site because of erythema and tenderness, but the
cultures of samples from the catheter are still pending. The patient recently started a new course of chemotherapy, which was administered through a
peripherally inserted central catheter (PICC) line in his left upper extremity. What laboratory tests should be performed? What organisms are likely to be
involved? What antibiotics should be administered?
Hint
A workup for sepsis should be performed.
Answer
Infected central-line site: Laboratory tests in this patient included assessments of the CBC with differential and electrolyte levels, 2 blood cultures, and
urine analysis. A chest radiograph was obtained. The goal is to isolate a potential source of infection other than the local cellulitis. After the blood
cultures were obtained, vancomycin therapy was started to cover Staphylococcus epidermidis and Staphylococcus aureus. Candida species are
occasionally involved; these cases require antifungal therapy. Leuconostoc species, which are treated with penicillin G or ampicillin, and lactobacillus,
which is treated with ampicillin and gentamicin, are rarely involved; these organisms are vancomycin resistant. This patient was neutropenic. He was
admitted to an isolation room. Reference: Gilbert DN, Moellering RC, Sande MA. The Sanford Guide to Antimicrobial Therapy. 30th ed. Hyde Park, VT:
Antimicrobial Therapy, Inc; 2000:46-9.

Swollen Eye

BACKGROUND
The patient is an otherwise healthy man who complains of swelling and irritation of his left eye. He stated that he touched the soil in his garden to check
the level of hydration and then rubbed his left eye. Within an hour, he experienced swelling and irritation. His uncorrected vision is 20/20 bilaterally.
What is the diagnosis? How should it be managed?
Hint
This patient has a type of edema seen in the eye.
Answer
Allergic conjunctivitis with chemosis: This patient has allergic conjunctivitis with chemosis, which is a diffuse swelling of the bulbar and fornical
conjunctiva. This sign can also be seen with chemical burns of the eye, traumatic retrobulbar hemorrhage, acute conjunctivitis, and allergic reactions.
Treatment is based on eliminating the inciting agent and on the application of cool compresses and topical drops. In mild cases, the use of artificial tears
(eg, Refresh) 4-8 times a day is sufficient. In moderate cases, the use of vasoconstrictor and antihistamine drops (eg, naphazoline with pheniramine) 4
times a day is indicated. In severe cases, the use of a topical steroid (eg, fluorometholone) 4 times a day for 1-2 weeks is recommended. Follow-up
treatment should occur in 1-2 weeks, or earlier if topical steroids are prescribed. Reference: Rhee DJ, Pyfer MF, eds. The Wills Eye Manual: Office and
Emergency Room Diagnosis and Treatment of Eye Disease. Wolters Kluwer Company; 1999.

Swollen Lower Lip

BACKGROUND
A 26-year-old woman who is 8 weeks pregnant presents with swelling of her lower lip and her legs and hands. The swelling began in the morning of the
day of presentation and is steadily worsening. She has not experienced any shortness of breath, lightheadedness, or nausea. Additionally, no wheezing
or stridor is noted on examination. She reports that she has had similar swelling in the past and that other members of her family also have episodes of
swelling in the extremities and face.
What is the diagnosis? How does her pregnancy affect her therapeutic options?
Hint
When tested, the patient^s C4 level is substantially lower than normal. She mentions that she has had multiple admissions to the intensive care unit
because of similar symptoms.
Answer
Hereditary angioedema (HAE): HAE is an autosomal dominant disorder resulting from a genetic deficiency of C1 esterase inhibitor protein (C1-INH).
There are 2 varieties of the disorder. Type 1 is most common and results from an abnormally low level of normal C1-INH. Type II results from normal or
abnormally elevated levels of a dysfunctional C1-INH. In both types of the disease, initial proteolytic components of the complement cascade (eg, C1r,
C1s) go relatively unopposed and lead to the characteristic presentation and laboratory abnormalities (eg, low levels of C2 and C4).
The disorder is characterized by recurrent episodes of localized, painless subcutaneous or submucosal edema. Symptoms may be mild or, in severe
cases, include incapacitating cutaneous swelling, gastrointestinal colic, and life-threatening upper airway obstruction requiring tracheostomy. The
episodes may have no clear precipitating event, or they may be initiated by minor trauma, anxiety, or instrumentation of the oropharynx. High estrogen
states, such as those observed during menstruation, pregnancy, or estrogen-containing contraception have been linked to increased severity of
episodes.
Treatment of acute episodes with standard agents used in anaphylactic or allergic reactions, such as epinephrine, antihistamines, and corticosteroids, is
of limited effectiveness. Fluid hydration and replacement of the consumed or deficient factors of the complement cascade with commercially available
C1-INH concentrate or fresh frozen plasma is the treatment of choice. Patients with evidence of progressive symptoms or those with systemic
manifestations or evidence of upper airway involvement should be observed and treated in an intensive care environment.
Prophylactic treatment is with androgen-containing agents, such as danazol and stanozolol. These agents help by increasing levels of C1-INH and later
components of the complement cascade such as C4. Unfortunately, these androgen derivatives are class X drugs, or those highly unsafe for use in
pregnancy because of a significantly increased risk of genital anomalies in newborns. Amicar (aminocaproic acid), which inhibits fibrinolysis, is another
therapeutic agent, but it is a class D drug, or one unsafe for use in pregnancy with evidence of risk that may be justifiable in certain clinical
circumstances.
In this case, an otolaryngologist was consulted, and on direct rhinolaryngoscopy, the patient^s airway was widely patent. As a precaution, she was
admitted to the medical intensive care unit for observation, and an allergy specialist was consulted. The patient was ultimately given fresh frozen
plasma, did not develop airway compromise, and had a good recovery from the episode.
For more information on HAE, see the eMedicine articles C1 Esterase Deficiency (within the Internal Medicine specialty) and Angioedema, Hereditary
(within the Dermatology specialty).

Swollen Toe of an Infant

BACKGROUND
This 3-month-old infant was brought to the emergency department by his parents, who noted about an hour ago that his right third toe was swollen. The
baby is crying vigorously and afebrile. His heart rate is 134 beats per minute, and his respiratory rate is 36 per minute. The patient had been born
prematurely at 36 weeks' gestation and has a twin. What is the diagnosis and how is it treated? What is the differential diagnosis?
Hint
The mother found a hair wrapped around the toe of the baby and removed it. On examination under magnification, another thin hair can be seen
wrapped around.
Answer
Hair-thread tourniquet syndrome: This is a classic case of hair-thread tourniquet syndrome in which fibers of hair or thread become tightly wrapped
around an appendage of an infant. The differential diagnosis includes ainhum, which is a condition of unknown etiology characterized by annular
constriction and spontaneous amputation of the fifth toe. Also in the differential diagnosis is pseudoainhum, which includes a group of conditions such
as congenital amnionic bands, bands secondary to specific diseases, and traumatic bands (eg, hair-thread tourniquet syndrome). The treatment is
focused on immediately finding and removing the constricting fiber. Surgical exploration is necessary if complete removal cannot be verified. In a 1988
review of the 66 cases in the literature, Barton et al found that the age range was from 20 days to 15 months, with a median age of 4 months. Most
cases involving the toes were caused by hair, whereas most cases involving the fingers were caused by thread from mittens. A few cases of labial or
penile strangulation were noted, providing yet another reason for physicians to routinely check the diapers of their pediatric patients. Only 2 of the 66
patients had complications, which included distal segment rotation and amputation. The appendages of the remaining patients healed well. In the
patient in this case, a digital block was performed, and the offending hair was successfully removed under magnification. General wound care
instructions and follow-up care were provided. Reference: Barton DJ, Sloan GM, Nichter LS, Reinisch JF: Hair-thread tourniquet syndrome. Pediatrics
1988 Dec; 82(6):925-8.

Tongue Lesion

BACKGROUND
What is this tongue lesion, and what is its significance?
Hint
What is the probable platelet count in this patient?
Answer
Hemorrhagic bulla of the tongue: A hemorrhagic bulla is pathognomonic for a platelet count of less than 5000 per cubic millimeter. Hemorrhagic bulla
occurs spontaneously, is often preceded by petechiae and bleeding, and usually occurs on the tongue or buccal mucosa. In this particular patient, a
platelet count of 2000 per cubic millimeter and idiopathic thrombocytopenic purpura (ITP) was diagnosed.

Treating Two Patients

BACKGROUND
A 26-year-old woman is being evaluated for symptoms of a cough, breathlessness, and chest tightness that began 4 weeks ago. She is 18 weeks'
gestation of her first pregnancy. The patient reports a history of respiratory difficulties in childhood that improved during adolescence. Physical
examination reveals mild congestion of the nasal mucosa, normal breath sounds, no wheezes or crackles, normal heart sounds, and no murmur.
Spirometry shows her forced expiratory volume in 1 second (FEV1) to be 2.4 L/s and her forced expiratory capacity (FVC) to be 3.6 L. After 2 puffs of
albuterol, FEV1 and FVC increase to 2.8 L/s and 3.8 L, respectively.
What is the diagnosis? Which medication will give her the best likelihood for a good perinatal outcome?
Hint
Approximately 30% of women with this pregnancy-related condition improve during term.
Answer
Asthma in pregnancy: This patient has possibly had a remote history of mild asthma, but she now has worsening of asthma in pregnancy. Her
symptoms are nonspecific; however, spirometry demonstrates reversible moderate airflow obstruction, as evidenced by a 16% improvement in FEV1
with albuterol. Asthma is diagnosed on the basis of the patient^s symptoms and spirometric findings. Among patients with asthma, the severity worsens
in 20-35% of pregnant patients, remains stable in 40-50%, and improves in 30%.
Asthma is a chronic inflammatory disease of the airways and is associated with airway constriction; airway edema; mucus plugging; and infiltration with
eosinophils, neutrophils, and lymphocytes. Leukotrienes, histamine, and cytokines trigger and mediate the airway inflammation via multiple pathways.
Asthma is a complication in 5% of pregnancies. Untreated, asthma in pregnancy has high rates of perinatal complications, such as perinatal fetal loss,
premature labor, preeclampsia, and low birth weight. Pregnant patients may present with atypical asthma symptoms, which can be mistaken for
symptoms of typical pregnancy-associated dyspnea. Physical examination may be unreliable in patients with asthma; therefore, an objective spirometric
assessment should be performed to confirm asthma and to assess the response to therapy. In severe asthma attacks, the fetal heart rate should also be
measured. A proactive approach to the management of asthma improves perinatal outcomes for the mother and the baby. Treatment goals in pregnant
patients with asthma are symptom resolution and optimization of FEV1 or peak expiratory flow rate (PEFR). Close follow-up during pregnancy and the
postpartum period should be arranged. A treatment summary is outlined in the image above.
The treatment of asthma depends on its severity, as reflected by the symptoms, use of reliever medications, and spirometric results. Supportive
measures include appropriate asthma education and avoidance of allergens, tobacco smoke, and other irritants. Prompt pharmacotherapy should be
directed toward symptom relief and control of airway inflammation. Short-acting b-agonists, such as albuterol, are prescribed on an as-needed basis; the
mainstay therapy is anti-inflammatory medication. The graphic above depicts asthma consensus guidelines to manage chronic asthma. These
guidelines are also generally used to treat pregnant patients with asthma.
Inhaled steroids are known to interrupt the inflammatory pathway by blocking the synthesis of mediators such as cytokines and leukotrienes. Several
studies have documented the efficacy of inhaled steroids in pregnancy; reductions in asthma exacerbations and improved perinatal outcomes are
reported. For pregnant asthma patients not treated with inhaled steroids, neonatal birth weights are 25% less than those of neonates born to treated
patients.
The inhaled steroids currently available in the United States are beclomethasone, triamcinolone, budesonide, flunisolide, and fluticasone. Safety data
are not available for all the inhaled steroids; however, budesonide is the only medication that is in pregnancy category B (ie, no evidence of risk in
humans).
Patients whose asthma is still uncontrolled after moderate-to-high doses of inhaled steroids may also require a long-acting b-agonist (eg, salmeterol,
formoterol), leukotriene receptor antagonists, or theophylline. The last 2 are also in pregnancy category B. Systemic corticosteroids are used for acute
exacerbations or at low doses for severe asthma unresponsive to other measures. Finally, all patients must be given a written asthma action plan, and
co-care with an internist, family physician, or pulmonologist is a requisite.

Unable to Sit

BACKGROUND
A 35-year-old woman presents to the hospital because she had a sudden onset of intense rectal pain 1 day ago and is unable to sit. She had painless
rectal bleeding weeks ago but did not seek medical attention. She also has been eating a low-fiber diet. What is the diagnosis and treatment?
Hint
The lesion is defined by its location below the dentate line. The procedure of choice is not exactly as shown in the photo. Follow-up care involves the
mnemonic WASH without the A.
Answer
Thrombosed external hemorrhoids: These lesions originate below the dentate line and receive their blood supply from the inferior hemorrhoidal plexus.
The hemorrhoids were excised under local anesthesia. An elliptical incision was made (the operator in this photo may have benefited by making a larger
ellipse), and the clot was evacuated; the procedure provided instant pain relief. Avoid simple incision and drainage in large thrombosed hemorrhoids
because rebleeding may occur and incomplete evacuation of the clot and residual skin tags may result. Appropriate follow-up instructions can be
remembered by using the mnemonic WASH, which stands for warm soaks (ie, sitz baths 3 times a day for 15 min), analgesics (which are unnecessary
after excision), stool softeners, and a high-fiber diet. Reference: Rosen P, ed. Emergency Medicine: Concepts and Clinical Practice. 4th ed. Mosby-Year
Book, Inc; 1998:2040.

Unusual Lesions of the Hand

BACKGROUND
A 46-year-old woman presents to the emergency department because of pain in the palm of her left hand. She began to feel intense burning pain in her
left arm about 7 days ago. Over the succeeding days, small blisters began to appear over her entire left arm, but these were most severe on her left
palm. The lesions were associated with a burning pain.
The patient treated her blisters with an over-the-counter antibiotic ointment and oral analgesics, but nothing seemed to help. Any manipulation or
contact with the blisters intensified the pain.
The lesions continued to grow and spread to the left side of her face and her left leg. The pain continued to worsen, prompting her to seek medical
attention. She reports having fevers but no trauma; burns; exposure to chemical agents; or exposure to any new medications, detergents, or cleaning
agents.
The patient's past medical and surgical history includes chronic lymphocytic leukemia, uterine fibroids, and menometrorrhagia, as well as abdominal
hysterectomy and bilateral calcaneal spur surgery. Her medications include Excedrin, and she has no known drug allergies. She stopped smoking
cigarettes several years ago and drinks alcohol only rarely. The review of systems is otherwise noncontributory.
Physical examination reveals the lesions, as well as a punctate, erythematous rash along the length of her left arm. She has a similar, though less
obvious, rash on the left sideon her back, face, and leg. Her arm is otherwise neurovascularly intact. She is afebrile, and her vital signs are within
normal limits.
What is the diagnosis?
Hint
Intense burning pain.
Answer
Herpes zoster (shingles): Herpes zoster is a viral infection caused by varicella-zoster virus (VZV), the same virus that causes chickenpox. First exposure
to the virus usually occurs in youth and most often manifests as childhood chickenpox. However, the virus can remain latent for years in the posterior
spinal ganglia or the cranial sensory ganglia and then become reactivated, manifesting as herpes zoster (shingles) in the adult.
Factors responsible for reactivating the virus are not well delineated but may include depression of the immune system (as occurred in this case
secondary to leukemia), trauma, radiation, medications, infection, or stress.
Herpes zoster is extremely common and mainly affects elderly or immunocompromised persons, but anyone who has had chickenpox is at risk. The
chief morbidity is intense, burning pain. Vesicles may also become superinfected, intensifying the pain. Mild cases may occur along an isolated
dermatomal distribution, but severe cases may become disseminated over several dermatomes.
Diagnosis is primarily based on the clinical history and physical examination, but laboratory tests and viral cultures may be useful in difficult cases.
Classic histologic findings are multinucleated giant cells with an accentuation of the nuclear material at the periphery of the nuclei.
The mainstay of treatment is antiviral therapy and analgesia. For disease distributed in a single dermatome in a patient without risk of CNS spread and
without ocular involvement or underlying immunosuppression, oral (PO) antiviral therapy decreases the pain and shortens the duration of lesions if
started within 48 hours of the onset of lesions. Many practitioners extend this window for PO therapy to 72 hours.
In patients with disseminated zoster, multidermatomal involvement, suspected or known CNS involvement, or relative immunosuppression, parenteral
acyclovir should be given instead, and the patient should be admitted to the hospital. Some clinicians still use steroids, which played a larger role in the
preantiviral era, to help manage the acute pain of shingles.
After the episode resolves, patients may have postherpetic neuralgia or visual problems if the globe is affected. Intracranial or internal manifestations
are well documented and may occur if the meninges or viscera are involved.
The patient in this case was treated with intravenous acyclovir, oral prednisone, narcotic analgesics, and topical antimicrobial ointment for areas of skin
breakdown.
For more information on herpes zoster, see the eMedicine articles Herpes Zoster (within the Internal Medicine specialty), Herpes Zoster (within the
Emergency Medicine specialty), and Herpes Zoster (within the Dermatology specialty).

Why Isn't This Child Growing?

BACKGROUND
A 3-year-old girl is brought to the pediatrician's office because she has been waking up nightly, at 2 AM, unable to breathe. The child struggles with her
breathing for several minutes before she is able to go back to sleep. She has had several of these episodes over the past few months. Her parents note
that she is not as active as before. They report no history of a cough or wheeze but have noticed that the child breathes rapidly during sleep.
Physical examination reveals that the child is hyperactive. Her thorax is barrel shaped, her breath sounds are reduced, and no wheezes or rhonchi are
present on auscultation. The child's growth chart is shown.
What is the diagnosis? How should this child be treated?
Hint
If the initial symptoms suggest a disease, that disease is probably the diagnosis.
Answer
Asthma, which should be treated with inhaled corticosteroids: This child^s symptoms are due to asthma, a respiratory disorder prevalent among 17
million Americans, of whom 30% are children. Experts say that if the initial symptoms suggest asthma, the patient probably has asthma. Recurrent
coughing, wheezing, dyspnea, rapid breathing, and chest tightness during exercise strongly suggest childhood asthma. Whenever possible, the
diagnosis should be confirmed with spirometric measurements.
The National Institutes of Health (NIH) has classified pediatric asthma on the basis of the frequency of symptoms and the variation in airflow
measurements. Types include mild intermittent, mild persistent, moderate persistent, and severe persistent.
The goal of treatment is to achieve good asthma control, which is defined as an ability to participate in normal activity, no need for acute care, the
presence of normal lung function, and the absence of symptoms. The 4 components of asthma treatment are (1) education, (2) assessment and
monitoring, (3) environmental control, and (4) pharmacotherapy.
Pharmacotherapy includes the use of inhaled corticosteroids, cromolyn and nedocromil sodium, oral corticosteroids, methylxanthines, leukotriene
antagonists, and long-acting beta2-agonists. A stepwise approach dependent on asthma severity is recommended. For moderate and severe persistent
asthma, medications of choice are inhaled corticosteroids alone or in combination with other medications. Inhaled corticosteroids are safe for use in
children of any age and have minimal systemic effects. The inhaled corticosteroids currently available in the United States are fluticasone, budesonide,
flunisolide, triamcinolone, and beclomethasone.
Alleviating parental concerns about the possible adverse effects of inhaled corticosteroids, especially their effect on growth, is important. Poorly
controlled asthma slows growth velocity. Clinical studies have shown that a short-term decrease in growth velocity occurs when treatment with
beclomethasone, budesonide, and fluticasone is initiated. However, several well-designed prospective, long-term studies have demonstrated that the
growth was not inhibited by these 3 inhaled corticosteroids.
Current recommendations are that the lowest possible doses of inhaled steroids should be used in children with asthma, with careful monitoring of their
height.
The child in this case was unable to use inhaled corticosteroids via a metered-dose inhaler (MDI) or a dry powder inhaler (DPI). Therefore, she was
given budesonide respules 0.25 mg twice daily via a jet nebulizer. At 6-month follow-up, the patient had no symptoms, and her growth returned to the
75th percentile.

Worm Found in a Diaper

BACKGROUND
An anxious father reports that he found a worm in the diaper of his 2-year-old son, who has had diarrhea for 2 days. The boy has no history of fever,
irritability, loss of appetite, or blood in the stool. The father states that the family has not traveled recently, but that his sister and her family, including
their 3-year-old child, have been visiting from Costa Rica for the past 6 weeks.
The child's vital signs are normal, with no abnormality on lung examination. The abdomen is soft, with no tenderness to palpation. The child generally
appears well and has no history of weight loss.
The worm was sent to pathology for identification. What is the diagnosis?
Hint
This nematode is the most common helminth.
Answer
Ascariasis: Ascaris lumbricoides is the cause of the most common helminthic infection worldwide. The infection is transmitted in a fecal-oral manner,
primarily from the ingestion of food contaminated with parasite eggs. Ascaris larvae hatch after ingestion and are released into the intestine, from where
they migrate to the lungs. After 1-2 weeks, the partially developed larvae ascend the trachea and are reswallowed. They then develop into mature
worms and produce eggs that are excreted in the stool. The worms can reach up to 30 cm in length.
This worm is thought to infect approximately 4 million Americans and an estimated 1.4 billion individuals worldwide. As in this case, the most common
presentation is the passage of a worm in the stool. Although most infected individuals are asymptomatic, some present with coughing, wheezing,
hemoptysis, colicky abdominal pain, or bilious emesis. Life-threatening complications may occur as a result of biliary or complete intestinal obstruction
due to a heavy worm burden. Ova and parasites may be found in the stool.
The drugs of choice are oral mebendazole 100 mg administered twice a day for 2 days or oral albendazole 400 mg given once, with another dose given
at 3 weeks. Flaccid paralyzing agents, such as piperazine citrate, are preferred when intestinal obstruction is suspected. Pyrantel pamoate, a spastic
paralyzing agent, is easier to administer as a single dose, but it increases the risk of intestinal obstruction.
For more information about ascariasis, see the eMedicine articles Ascariasis (within the Pediatrics specialty) and Ascariasis and Nematode Infections
(within the Internal Medicine specialty).

Você também pode gostar